2021 AMC 12A Problems/Problem 1

Revision as of 03:34, 31 January 2021 by Jhu08 (talk | contribs) (Problem)

So assume that 1 + 1 = 3

Using this logic, prove that: looking + at + this + problem = you're a cheater.

Solution

The solutions will be posted once the problems are posted.

See also

2021 AMC 12A (ProblemsAnswer KeyResources)
Preceded by
First problem
Followed by
Problem 2
1 2 3 4 5 6 7 8 9 10 11 12 13 14 15 16 17 18 19 20 21 22 23 24 25
All AMC 12 Problems and Solutions

The problems on this page are copyrighted by the Mathematical Association of America's American Mathematics Competitions. AMC logo.png